Proof Verification: Differentiability implies continuity.











up vote
3
down vote

favorite
1












Proof:



Let $x$ be a real number, since $f(x)$ is differentiable, or $lim_{x to
a} frac{f(x)-f(a)}{x-a}$
exists and is equal to $f'(a)$. So, for any $epsilon' > 0$ there exists a $delta > 0$, such that $0<|x-a| < delta$



$implies |frac{f(x)-f(a)}{x-a} - f'(a)| < epsilon'$



Using the Triangle inequality, we get



$ |frac{f(x)-f(a)}{x-a}| < epsilon' + |f'(a)|$



$implies |f(x)-f(a)| < epsilon' delta + |f'(a)| delta$



Here's where I start getting unsure whether my proof is correct or not. This inequality is true for $epsilon'$ = $frac{epsilon}{delta'} - |f'(a)|$ where $delta' < delta$ such that $frac{epsilon}{delta'} > |f'(a)|$



This gives us



$ |f(x)-f(a)| < epsilon$



The logic behind the proof is that if someone gives asks me to find an appropriate delta for $epsilon = h$, I'll find a delta for $epsilon' = h$, say that delta is equal to $n$. Next I solve for $epsilon'$ using $epsilon' = frac{h}{n} - |f'(a)|$. If on solving I get a negative $epsilon'$, I decrease $delta'$ to a value $n_0$ such that I get a positive value for $epsilon'$ and then get the corresponding delta, $n_1$. The minimum of $n_1$ and $n_0$ would be the required delta.



Please let me know where exactly the proof starts to go wrong. While alternate proofs are appreciated, the main goal here is understanding why this proof is wrong.



EDIT: After thinking about the problem for a while and reading a few of the answers, I found a way to communicate the idea more effectively.



Continuing from $|f(x)-f(a)| < (epsilon' + |f'(a)|) delta$



Now we need to show that we can represent any positive real number, $epsilon$, by taking appropriate values of $delta$ and $epsilon'$. Fix $epsilon' = 1$. Now $delta$ is just $frac{epsilon}{1+|f'(a)|}$. To complete our answer, let one value of $delta$ for $epsilon' = 1$ be $delta_0$. Our final answer would be $delta = min(delta_0,frac{epsilon}{1+|f'(a)|})$










share|cite|improve this question
























  • There are some errors.. the derivative must be calculate on a and not in x
    – Federico Fallucca
    Nov 19 at 14:53












  • Fixed, thanks. Is the general idea of the proof correct? Because I haven't really seen the argument I used at the end in any other proof, I'm not sure if it's correct or not.
    – Star Platinum ZA WARUDO
    Nov 19 at 15:05










  • No, I think it is wrong because you use the fixed constant epsilon and delta to get the result but I guess that you must use only the variable x.
    – Federico Fallucca
    Nov 19 at 15:11















up vote
3
down vote

favorite
1












Proof:



Let $x$ be a real number, since $f(x)$ is differentiable, or $lim_{x to
a} frac{f(x)-f(a)}{x-a}$
exists and is equal to $f'(a)$. So, for any $epsilon' > 0$ there exists a $delta > 0$, such that $0<|x-a| < delta$



$implies |frac{f(x)-f(a)}{x-a} - f'(a)| < epsilon'$



Using the Triangle inequality, we get



$ |frac{f(x)-f(a)}{x-a}| < epsilon' + |f'(a)|$



$implies |f(x)-f(a)| < epsilon' delta + |f'(a)| delta$



Here's where I start getting unsure whether my proof is correct or not. This inequality is true for $epsilon'$ = $frac{epsilon}{delta'} - |f'(a)|$ where $delta' < delta$ such that $frac{epsilon}{delta'} > |f'(a)|$



This gives us



$ |f(x)-f(a)| < epsilon$



The logic behind the proof is that if someone gives asks me to find an appropriate delta for $epsilon = h$, I'll find a delta for $epsilon' = h$, say that delta is equal to $n$. Next I solve for $epsilon'$ using $epsilon' = frac{h}{n} - |f'(a)|$. If on solving I get a negative $epsilon'$, I decrease $delta'$ to a value $n_0$ such that I get a positive value for $epsilon'$ and then get the corresponding delta, $n_1$. The minimum of $n_1$ and $n_0$ would be the required delta.



Please let me know where exactly the proof starts to go wrong. While alternate proofs are appreciated, the main goal here is understanding why this proof is wrong.



EDIT: After thinking about the problem for a while and reading a few of the answers, I found a way to communicate the idea more effectively.



Continuing from $|f(x)-f(a)| < (epsilon' + |f'(a)|) delta$



Now we need to show that we can represent any positive real number, $epsilon$, by taking appropriate values of $delta$ and $epsilon'$. Fix $epsilon' = 1$. Now $delta$ is just $frac{epsilon}{1+|f'(a)|}$. To complete our answer, let one value of $delta$ for $epsilon' = 1$ be $delta_0$. Our final answer would be $delta = min(delta_0,frac{epsilon}{1+|f'(a)|})$










share|cite|improve this question
























  • There are some errors.. the derivative must be calculate on a and not in x
    – Federico Fallucca
    Nov 19 at 14:53












  • Fixed, thanks. Is the general idea of the proof correct? Because I haven't really seen the argument I used at the end in any other proof, I'm not sure if it's correct or not.
    – Star Platinum ZA WARUDO
    Nov 19 at 15:05










  • No, I think it is wrong because you use the fixed constant epsilon and delta to get the result but I guess that you must use only the variable x.
    – Federico Fallucca
    Nov 19 at 15:11













up vote
3
down vote

favorite
1









up vote
3
down vote

favorite
1






1





Proof:



Let $x$ be a real number, since $f(x)$ is differentiable, or $lim_{x to
a} frac{f(x)-f(a)}{x-a}$
exists and is equal to $f'(a)$. So, for any $epsilon' > 0$ there exists a $delta > 0$, such that $0<|x-a| < delta$



$implies |frac{f(x)-f(a)}{x-a} - f'(a)| < epsilon'$



Using the Triangle inequality, we get



$ |frac{f(x)-f(a)}{x-a}| < epsilon' + |f'(a)|$



$implies |f(x)-f(a)| < epsilon' delta + |f'(a)| delta$



Here's where I start getting unsure whether my proof is correct or not. This inequality is true for $epsilon'$ = $frac{epsilon}{delta'} - |f'(a)|$ where $delta' < delta$ such that $frac{epsilon}{delta'} > |f'(a)|$



This gives us



$ |f(x)-f(a)| < epsilon$



The logic behind the proof is that if someone gives asks me to find an appropriate delta for $epsilon = h$, I'll find a delta for $epsilon' = h$, say that delta is equal to $n$. Next I solve for $epsilon'$ using $epsilon' = frac{h}{n} - |f'(a)|$. If on solving I get a negative $epsilon'$, I decrease $delta'$ to a value $n_0$ such that I get a positive value for $epsilon'$ and then get the corresponding delta, $n_1$. The minimum of $n_1$ and $n_0$ would be the required delta.



Please let me know where exactly the proof starts to go wrong. While alternate proofs are appreciated, the main goal here is understanding why this proof is wrong.



EDIT: After thinking about the problem for a while and reading a few of the answers, I found a way to communicate the idea more effectively.



Continuing from $|f(x)-f(a)| < (epsilon' + |f'(a)|) delta$



Now we need to show that we can represent any positive real number, $epsilon$, by taking appropriate values of $delta$ and $epsilon'$. Fix $epsilon' = 1$. Now $delta$ is just $frac{epsilon}{1+|f'(a)|}$. To complete our answer, let one value of $delta$ for $epsilon' = 1$ be $delta_0$. Our final answer would be $delta = min(delta_0,frac{epsilon}{1+|f'(a)|})$










share|cite|improve this question















Proof:



Let $x$ be a real number, since $f(x)$ is differentiable, or $lim_{x to
a} frac{f(x)-f(a)}{x-a}$
exists and is equal to $f'(a)$. So, for any $epsilon' > 0$ there exists a $delta > 0$, such that $0<|x-a| < delta$



$implies |frac{f(x)-f(a)}{x-a} - f'(a)| < epsilon'$



Using the Triangle inequality, we get



$ |frac{f(x)-f(a)}{x-a}| < epsilon' + |f'(a)|$



$implies |f(x)-f(a)| < epsilon' delta + |f'(a)| delta$



Here's where I start getting unsure whether my proof is correct or not. This inequality is true for $epsilon'$ = $frac{epsilon}{delta'} - |f'(a)|$ where $delta' < delta$ such that $frac{epsilon}{delta'} > |f'(a)|$



This gives us



$ |f(x)-f(a)| < epsilon$



The logic behind the proof is that if someone gives asks me to find an appropriate delta for $epsilon = h$, I'll find a delta for $epsilon' = h$, say that delta is equal to $n$. Next I solve for $epsilon'$ using $epsilon' = frac{h}{n} - |f'(a)|$. If on solving I get a negative $epsilon'$, I decrease $delta'$ to a value $n_0$ such that I get a positive value for $epsilon'$ and then get the corresponding delta, $n_1$. The minimum of $n_1$ and $n_0$ would be the required delta.



Please let me know where exactly the proof starts to go wrong. While alternate proofs are appreciated, the main goal here is understanding why this proof is wrong.



EDIT: After thinking about the problem for a while and reading a few of the answers, I found a way to communicate the idea more effectively.



Continuing from $|f(x)-f(a)| < (epsilon' + |f'(a)|) delta$



Now we need to show that we can represent any positive real number, $epsilon$, by taking appropriate values of $delta$ and $epsilon'$. Fix $epsilon' = 1$. Now $delta$ is just $frac{epsilon}{1+|f'(a)|}$. To complete our answer, let one value of $delta$ for $epsilon' = 1$ be $delta_0$. Our final answer would be $delta = min(delta_0,frac{epsilon}{1+|f'(a)|})$







real-analysis epsilon-delta






share|cite|improve this question















share|cite|improve this question













share|cite|improve this question




share|cite|improve this question








edited Nov 20 at 1:42









miracle173

7,31222247




7,31222247










asked Nov 19 at 14:50









Star Platinum ZA WARUDO

33412




33412












  • There are some errors.. the derivative must be calculate on a and not in x
    – Federico Fallucca
    Nov 19 at 14:53












  • Fixed, thanks. Is the general idea of the proof correct? Because I haven't really seen the argument I used at the end in any other proof, I'm not sure if it's correct or not.
    – Star Platinum ZA WARUDO
    Nov 19 at 15:05










  • No, I think it is wrong because you use the fixed constant epsilon and delta to get the result but I guess that you must use only the variable x.
    – Federico Fallucca
    Nov 19 at 15:11


















  • There are some errors.. the derivative must be calculate on a and not in x
    – Federico Fallucca
    Nov 19 at 14:53












  • Fixed, thanks. Is the general idea of the proof correct? Because I haven't really seen the argument I used at the end in any other proof, I'm not sure if it's correct or not.
    – Star Platinum ZA WARUDO
    Nov 19 at 15:05










  • No, I think it is wrong because you use the fixed constant epsilon and delta to get the result but I guess that you must use only the variable x.
    – Federico Fallucca
    Nov 19 at 15:11
















There are some errors.. the derivative must be calculate on a and not in x
– Federico Fallucca
Nov 19 at 14:53






There are some errors.. the derivative must be calculate on a and not in x
– Federico Fallucca
Nov 19 at 14:53














Fixed, thanks. Is the general idea of the proof correct? Because I haven't really seen the argument I used at the end in any other proof, I'm not sure if it's correct or not.
– Star Platinum ZA WARUDO
Nov 19 at 15:05




Fixed, thanks. Is the general idea of the proof correct? Because I haven't really seen the argument I used at the end in any other proof, I'm not sure if it's correct or not.
– Star Platinum ZA WARUDO
Nov 19 at 15:05












No, I think it is wrong because you use the fixed constant epsilon and delta to get the result but I guess that you must use only the variable x.
– Federico Fallucca
Nov 19 at 15:11




No, I think it is wrong because you use the fixed constant epsilon and delta to get the result but I guess that you must use only the variable x.
– Federico Fallucca
Nov 19 at 15:11










6 Answers
6






active

oldest

votes

















up vote
0
down vote



accepted










I try to put your arguments in the right order.



Proof:



Assume that $f$ is differentiable in $a$. So $f'(a)$ exists.



To prove that $f$ is continous in $a$ choose an arbitrary $varepsilon>0.$ Now you can choose $delta_1$ such
$$0<delta_1<frac{varepsilon}{|f'(a)|}, text{ if } f'(a)ne 0$$
$$delta_1=1, text{ if } |f'(a)|=0.$$
So we have
$$0<|f'(a)|<frac{varepsilon}{delta_1}$$
and define
$$varepsilon_2=frac{varepsilon}{delta_1}-|f'(a)|>0$$



Because f is differentiable in $a$ we can find a $delta_2$ such that
$$|frac{f(x)-f(a)}{x-a} - f'(a)| < varepsilon_2,; forall x: 0<|x-a|<delta_2$$



Using the triangle inequality, we get



$$ |frac{f(x)-f(a)}{x-a}| < varepsilon_2 + |f'(a)|,; forall x: 0<|x-a|<min(delta_1,delta_2)$$



So we set
$$delta=min(delta_1,delta_2)$$
and get
$$| f(x)-f(a)| < (varepsilon_2 + |f'(a)|) delta_2<frac{varepsilon}{delta_1}min(delta_1,delta_2)le varepsilon,; forall x: 0<|x-a|<delta$$






share|cite|improve this answer






























    up vote
    7
    down vote













    Or just see that
    $$lim_{hrightarrow 0}f(x+h)-f(x)=lim_{hrightarrow 0}underbrace{frac{f(x+h)-f(x)}{h}}_{rightarrow f'(x)}h=0.$$






    share|cite|improve this answer

















    • 2




      Nice job latexing.
      – djechlin
      Nov 19 at 18:36


















    up vote
    2
    down vote













    We can simply use the equivalent definition of differentiability



    $$f(a+h)=f(a)+f'(a)cdot h +o(h) implies lim_{xto a} f(x)=lim_{hto 0} f(a+h)=f(a)$$






    share|cite|improve this answer





















    • Thanks for the answer. I know my proof is long and a bit harder to read, but is it correct? If it isn't, where exactly does it go wrong?
      – Star Platinum ZA WARUDO
      Nov 19 at 15:03










    • @StarPlatinumZAWARUDO It seems really over complicated to me, the implication is avery simple ansd trivial fact. I'll try to look to it more carefully later. Bye
      – gimusi
      Nov 19 at 15:10


















    up vote
    1
    down vote













    $ |frac{f(x)-f(a)}{x-a}| < epsilon' + |f'(a)|$



    so



    $ |f(x)-f(a)| <( epsilon' + |f'(a)|)|x-a|$



    then for $xto a$ you have that



    $|f(x)-f(a)|leq 0$






    share|cite|improve this answer




























      up vote
      1
      down vote













      The heart of your argument is basically the following. Give me some $epsilon$, and I have to find some $delta$ so that $|f(x)-f(a)|<epsilon$ when $|x-a|<delta$. To start with, I'll just take any $delta$, it doesn't matter. Now, within the range $[a-delta, a+delta]$, the function $xtofrac{f(x)-f(a)}{x-a}$ is bounded, because it's convergent at $a$ and convergent functions are locally bounded - this is a theorem which you essentially spend the first couple of lines of your proof re-proving by way of the value $epsilon'$. So say $lvertfrac{f(x)-f(a)}{x-a}rvert<M$ on the interval $[a-delta, a+delta]$. Well then certainly $|f(x)-f(a)|<Mdelta$, and by choosing a sufficiently small $delta$ we can make that less than $epsilon$ (since the $M$ only gets smaller as $delta$ gets smaller).



      If you notice, in your own proof, the $epsilon'$ is a bit pointless. Since at the end you're just going to say "and now take $delta$ as small as is necessary to make this true", you may as well initially take $epsilon'=10^{100}$. All that matters is that $epsilon'$ is finite and that $delta$ can be made arbitrarily small, in other words, that the difference quotient is locally bounded. The exact value of the derivative at $a$ also doesn't matter.






      share|cite|improve this answer




























        up vote
        1
        down vote













        Not quite right. One of the issues is:




        I define $epsilon'$ as $frac epsilon delta $




        As it stands, this doesn't make sense, because you started the proof by taking an arbitrary $epsilon'$. Perhaps you mean something like: Since this inequality is true for any $epsilon '$, it is true in particular for $frac epsilon delta$ ...



        But this doesn't quite make sense either; What's the $delta$ on the right-hand side? This $delta $ may depend on the $epsilon '$ you initially choose.





        Try this alternative:



        For any $epsilon> 0$ there is a $delta' > 0$, such that whenever $0 < |x-a| < delta'$, one has $left|frac{f(x)-f(a)}{x-a}right| < |f'(a)| + epsilon$. In particular, there is a $delta_0>0$ such that $left|frac{f(x)-f(a)}{x-a}right| < |f'(a)| + 1$



        Now, choose $delta = min left(delta_0, frac 1 {|f'(a)+1|}right) $



        Can you complete the proof from here?






        share|cite|improve this answer























        • I've made a few edits to the ending paragraphs to make it a bit easier to understand what I'm trying to say.
          – Star Platinum ZA WARUDO
          Nov 19 at 15:44











        Your Answer





        StackExchange.ifUsing("editor", function () {
        return StackExchange.using("mathjaxEditing", function () {
        StackExchange.MarkdownEditor.creationCallbacks.add(function (editor, postfix) {
        StackExchange.mathjaxEditing.prepareWmdForMathJax(editor, postfix, [["$", "$"], ["\\(","\\)"]]);
        });
        });
        }, "mathjax-editing");

        StackExchange.ready(function() {
        var channelOptions = {
        tags: "".split(" "),
        id: "69"
        };
        initTagRenderer("".split(" "), "".split(" "), channelOptions);

        StackExchange.using("externalEditor", function() {
        // Have to fire editor after snippets, if snippets enabled
        if (StackExchange.settings.snippets.snippetsEnabled) {
        StackExchange.using("snippets", function() {
        createEditor();
        });
        }
        else {
        createEditor();
        }
        });

        function createEditor() {
        StackExchange.prepareEditor({
        heartbeatType: 'answer',
        convertImagesToLinks: true,
        noModals: true,
        showLowRepImageUploadWarning: true,
        reputationToPostImages: 10,
        bindNavPrevention: true,
        postfix: "",
        imageUploader: {
        brandingHtml: "Powered by u003ca class="icon-imgur-white" href="https://imgur.com/"u003eu003c/au003e",
        contentPolicyHtml: "User contributions licensed under u003ca href="https://creativecommons.org/licenses/by-sa/3.0/"u003ecc by-sa 3.0 with attribution requiredu003c/au003e u003ca href="https://stackoverflow.com/legal/content-policy"u003e(content policy)u003c/au003e",
        allowUrls: true
        },
        noCode: true, onDemand: true,
        discardSelector: ".discard-answer"
        ,immediatelyShowMarkdownHelp:true
        });


        }
        });














        draft saved

        draft discarded


















        StackExchange.ready(
        function () {
        StackExchange.openid.initPostLogin('.new-post-login', 'https%3a%2f%2fmath.stackexchange.com%2fquestions%2f3005021%2fproof-verification-differentiability-implies-continuity%23new-answer', 'question_page');
        }
        );

        Post as a guest















        Required, but never shown

























        6 Answers
        6






        active

        oldest

        votes








        6 Answers
        6






        active

        oldest

        votes









        active

        oldest

        votes






        active

        oldest

        votes








        up vote
        0
        down vote



        accepted










        I try to put your arguments in the right order.



        Proof:



        Assume that $f$ is differentiable in $a$. So $f'(a)$ exists.



        To prove that $f$ is continous in $a$ choose an arbitrary $varepsilon>0.$ Now you can choose $delta_1$ such
        $$0<delta_1<frac{varepsilon}{|f'(a)|}, text{ if } f'(a)ne 0$$
        $$delta_1=1, text{ if } |f'(a)|=0.$$
        So we have
        $$0<|f'(a)|<frac{varepsilon}{delta_1}$$
        and define
        $$varepsilon_2=frac{varepsilon}{delta_1}-|f'(a)|>0$$



        Because f is differentiable in $a$ we can find a $delta_2$ such that
        $$|frac{f(x)-f(a)}{x-a} - f'(a)| < varepsilon_2,; forall x: 0<|x-a|<delta_2$$



        Using the triangle inequality, we get



        $$ |frac{f(x)-f(a)}{x-a}| < varepsilon_2 + |f'(a)|,; forall x: 0<|x-a|<min(delta_1,delta_2)$$



        So we set
        $$delta=min(delta_1,delta_2)$$
        and get
        $$| f(x)-f(a)| < (varepsilon_2 + |f'(a)|) delta_2<frac{varepsilon}{delta_1}min(delta_1,delta_2)le varepsilon,; forall x: 0<|x-a|<delta$$






        share|cite|improve this answer



























          up vote
          0
          down vote



          accepted










          I try to put your arguments in the right order.



          Proof:



          Assume that $f$ is differentiable in $a$. So $f'(a)$ exists.



          To prove that $f$ is continous in $a$ choose an arbitrary $varepsilon>0.$ Now you can choose $delta_1$ such
          $$0<delta_1<frac{varepsilon}{|f'(a)|}, text{ if } f'(a)ne 0$$
          $$delta_1=1, text{ if } |f'(a)|=0.$$
          So we have
          $$0<|f'(a)|<frac{varepsilon}{delta_1}$$
          and define
          $$varepsilon_2=frac{varepsilon}{delta_1}-|f'(a)|>0$$



          Because f is differentiable in $a$ we can find a $delta_2$ such that
          $$|frac{f(x)-f(a)}{x-a} - f'(a)| < varepsilon_2,; forall x: 0<|x-a|<delta_2$$



          Using the triangle inequality, we get



          $$ |frac{f(x)-f(a)}{x-a}| < varepsilon_2 + |f'(a)|,; forall x: 0<|x-a|<min(delta_1,delta_2)$$



          So we set
          $$delta=min(delta_1,delta_2)$$
          and get
          $$| f(x)-f(a)| < (varepsilon_2 + |f'(a)|) delta_2<frac{varepsilon}{delta_1}min(delta_1,delta_2)le varepsilon,; forall x: 0<|x-a|<delta$$






          share|cite|improve this answer

























            up vote
            0
            down vote



            accepted







            up vote
            0
            down vote



            accepted






            I try to put your arguments in the right order.



            Proof:



            Assume that $f$ is differentiable in $a$. So $f'(a)$ exists.



            To prove that $f$ is continous in $a$ choose an arbitrary $varepsilon>0.$ Now you can choose $delta_1$ such
            $$0<delta_1<frac{varepsilon}{|f'(a)|}, text{ if } f'(a)ne 0$$
            $$delta_1=1, text{ if } |f'(a)|=0.$$
            So we have
            $$0<|f'(a)|<frac{varepsilon}{delta_1}$$
            and define
            $$varepsilon_2=frac{varepsilon}{delta_1}-|f'(a)|>0$$



            Because f is differentiable in $a$ we can find a $delta_2$ such that
            $$|frac{f(x)-f(a)}{x-a} - f'(a)| < varepsilon_2,; forall x: 0<|x-a|<delta_2$$



            Using the triangle inequality, we get



            $$ |frac{f(x)-f(a)}{x-a}| < varepsilon_2 + |f'(a)|,; forall x: 0<|x-a|<min(delta_1,delta_2)$$



            So we set
            $$delta=min(delta_1,delta_2)$$
            and get
            $$| f(x)-f(a)| < (varepsilon_2 + |f'(a)|) delta_2<frac{varepsilon}{delta_1}min(delta_1,delta_2)le varepsilon,; forall x: 0<|x-a|<delta$$






            share|cite|improve this answer














            I try to put your arguments in the right order.



            Proof:



            Assume that $f$ is differentiable in $a$. So $f'(a)$ exists.



            To prove that $f$ is continous in $a$ choose an arbitrary $varepsilon>0.$ Now you can choose $delta_1$ such
            $$0<delta_1<frac{varepsilon}{|f'(a)|}, text{ if } f'(a)ne 0$$
            $$delta_1=1, text{ if } |f'(a)|=0.$$
            So we have
            $$0<|f'(a)|<frac{varepsilon}{delta_1}$$
            and define
            $$varepsilon_2=frac{varepsilon}{delta_1}-|f'(a)|>0$$



            Because f is differentiable in $a$ we can find a $delta_2$ such that
            $$|frac{f(x)-f(a)}{x-a} - f'(a)| < varepsilon_2,; forall x: 0<|x-a|<delta_2$$



            Using the triangle inequality, we get



            $$ |frac{f(x)-f(a)}{x-a}| < varepsilon_2 + |f'(a)|,; forall x: 0<|x-a|<min(delta_1,delta_2)$$



            So we set
            $$delta=min(delta_1,delta_2)$$
            and get
            $$| f(x)-f(a)| < (varepsilon_2 + |f'(a)|) delta_2<frac{varepsilon}{delta_1}min(delta_1,delta_2)le varepsilon,; forall x: 0<|x-a|<delta$$







            share|cite|improve this answer














            share|cite|improve this answer



            share|cite|improve this answer








            edited Nov 20 at 7:29

























            answered Nov 20 at 1:40









            miracle173

            7,31222247




            7,31222247






















                up vote
                7
                down vote













                Or just see that
                $$lim_{hrightarrow 0}f(x+h)-f(x)=lim_{hrightarrow 0}underbrace{frac{f(x+h)-f(x)}{h}}_{rightarrow f'(x)}h=0.$$






                share|cite|improve this answer

















                • 2




                  Nice job latexing.
                  – djechlin
                  Nov 19 at 18:36















                up vote
                7
                down vote













                Or just see that
                $$lim_{hrightarrow 0}f(x+h)-f(x)=lim_{hrightarrow 0}underbrace{frac{f(x+h)-f(x)}{h}}_{rightarrow f'(x)}h=0.$$






                share|cite|improve this answer

















                • 2




                  Nice job latexing.
                  – djechlin
                  Nov 19 at 18:36













                up vote
                7
                down vote










                up vote
                7
                down vote









                Or just see that
                $$lim_{hrightarrow 0}f(x+h)-f(x)=lim_{hrightarrow 0}underbrace{frac{f(x+h)-f(x)}{h}}_{rightarrow f'(x)}h=0.$$






                share|cite|improve this answer












                Or just see that
                $$lim_{hrightarrow 0}f(x+h)-f(x)=lim_{hrightarrow 0}underbrace{frac{f(x+h)-f(x)}{h}}_{rightarrow f'(x)}h=0.$$







                share|cite|improve this answer












                share|cite|improve this answer



                share|cite|improve this answer










                answered Nov 19 at 15:01









                Peter Melech

                2,529813




                2,529813








                • 2




                  Nice job latexing.
                  – djechlin
                  Nov 19 at 18:36














                • 2




                  Nice job latexing.
                  – djechlin
                  Nov 19 at 18:36








                2




                2




                Nice job latexing.
                – djechlin
                Nov 19 at 18:36




                Nice job latexing.
                – djechlin
                Nov 19 at 18:36










                up vote
                2
                down vote













                We can simply use the equivalent definition of differentiability



                $$f(a+h)=f(a)+f'(a)cdot h +o(h) implies lim_{xto a} f(x)=lim_{hto 0} f(a+h)=f(a)$$






                share|cite|improve this answer





















                • Thanks for the answer. I know my proof is long and a bit harder to read, but is it correct? If it isn't, where exactly does it go wrong?
                  – Star Platinum ZA WARUDO
                  Nov 19 at 15:03










                • @StarPlatinumZAWARUDO It seems really over complicated to me, the implication is avery simple ansd trivial fact. I'll try to look to it more carefully later. Bye
                  – gimusi
                  Nov 19 at 15:10















                up vote
                2
                down vote













                We can simply use the equivalent definition of differentiability



                $$f(a+h)=f(a)+f'(a)cdot h +o(h) implies lim_{xto a} f(x)=lim_{hto 0} f(a+h)=f(a)$$






                share|cite|improve this answer





















                • Thanks for the answer. I know my proof is long and a bit harder to read, but is it correct? If it isn't, where exactly does it go wrong?
                  – Star Platinum ZA WARUDO
                  Nov 19 at 15:03










                • @StarPlatinumZAWARUDO It seems really over complicated to me, the implication is avery simple ansd trivial fact. I'll try to look to it more carefully later. Bye
                  – gimusi
                  Nov 19 at 15:10













                up vote
                2
                down vote










                up vote
                2
                down vote









                We can simply use the equivalent definition of differentiability



                $$f(a+h)=f(a)+f'(a)cdot h +o(h) implies lim_{xto a} f(x)=lim_{hto 0} f(a+h)=f(a)$$






                share|cite|improve this answer












                We can simply use the equivalent definition of differentiability



                $$f(a+h)=f(a)+f'(a)cdot h +o(h) implies lim_{xto a} f(x)=lim_{hto 0} f(a+h)=f(a)$$







                share|cite|improve this answer












                share|cite|improve this answer



                share|cite|improve this answer










                answered Nov 19 at 14:54









                gimusi

                91.7k84495




                91.7k84495












                • Thanks for the answer. I know my proof is long and a bit harder to read, but is it correct? If it isn't, where exactly does it go wrong?
                  – Star Platinum ZA WARUDO
                  Nov 19 at 15:03










                • @StarPlatinumZAWARUDO It seems really over complicated to me, the implication is avery simple ansd trivial fact. I'll try to look to it more carefully later. Bye
                  – gimusi
                  Nov 19 at 15:10


















                • Thanks for the answer. I know my proof is long and a bit harder to read, but is it correct? If it isn't, where exactly does it go wrong?
                  – Star Platinum ZA WARUDO
                  Nov 19 at 15:03










                • @StarPlatinumZAWARUDO It seems really over complicated to me, the implication is avery simple ansd trivial fact. I'll try to look to it more carefully later. Bye
                  – gimusi
                  Nov 19 at 15:10
















                Thanks for the answer. I know my proof is long and a bit harder to read, but is it correct? If it isn't, where exactly does it go wrong?
                – Star Platinum ZA WARUDO
                Nov 19 at 15:03




                Thanks for the answer. I know my proof is long and a bit harder to read, but is it correct? If it isn't, where exactly does it go wrong?
                – Star Platinum ZA WARUDO
                Nov 19 at 15:03












                @StarPlatinumZAWARUDO It seems really over complicated to me, the implication is avery simple ansd trivial fact. I'll try to look to it more carefully later. Bye
                – gimusi
                Nov 19 at 15:10




                @StarPlatinumZAWARUDO It seems really over complicated to me, the implication is avery simple ansd trivial fact. I'll try to look to it more carefully later. Bye
                – gimusi
                Nov 19 at 15:10










                up vote
                1
                down vote













                $ |frac{f(x)-f(a)}{x-a}| < epsilon' + |f'(a)|$



                so



                $ |f(x)-f(a)| <( epsilon' + |f'(a)|)|x-a|$



                then for $xto a$ you have that



                $|f(x)-f(a)|leq 0$






                share|cite|improve this answer

























                  up vote
                  1
                  down vote













                  $ |frac{f(x)-f(a)}{x-a}| < epsilon' + |f'(a)|$



                  so



                  $ |f(x)-f(a)| <( epsilon' + |f'(a)|)|x-a|$



                  then for $xto a$ you have that



                  $|f(x)-f(a)|leq 0$






                  share|cite|improve this answer























                    up vote
                    1
                    down vote










                    up vote
                    1
                    down vote









                    $ |frac{f(x)-f(a)}{x-a}| < epsilon' + |f'(a)|$



                    so



                    $ |f(x)-f(a)| <( epsilon' + |f'(a)|)|x-a|$



                    then for $xto a$ you have that



                    $|f(x)-f(a)|leq 0$






                    share|cite|improve this answer












                    $ |frac{f(x)-f(a)}{x-a}| < epsilon' + |f'(a)|$



                    so



                    $ |f(x)-f(a)| <( epsilon' + |f'(a)|)|x-a|$



                    then for $xto a$ you have that



                    $|f(x)-f(a)|leq 0$







                    share|cite|improve this answer












                    share|cite|improve this answer



                    share|cite|improve this answer










                    answered Nov 19 at 14:57









                    Federico Fallucca

                    1,74318




                    1,74318






















                        up vote
                        1
                        down vote













                        The heart of your argument is basically the following. Give me some $epsilon$, and I have to find some $delta$ so that $|f(x)-f(a)|<epsilon$ when $|x-a|<delta$. To start with, I'll just take any $delta$, it doesn't matter. Now, within the range $[a-delta, a+delta]$, the function $xtofrac{f(x)-f(a)}{x-a}$ is bounded, because it's convergent at $a$ and convergent functions are locally bounded - this is a theorem which you essentially spend the first couple of lines of your proof re-proving by way of the value $epsilon'$. So say $lvertfrac{f(x)-f(a)}{x-a}rvert<M$ on the interval $[a-delta, a+delta]$. Well then certainly $|f(x)-f(a)|<Mdelta$, and by choosing a sufficiently small $delta$ we can make that less than $epsilon$ (since the $M$ only gets smaller as $delta$ gets smaller).



                        If you notice, in your own proof, the $epsilon'$ is a bit pointless. Since at the end you're just going to say "and now take $delta$ as small as is necessary to make this true", you may as well initially take $epsilon'=10^{100}$. All that matters is that $epsilon'$ is finite and that $delta$ can be made arbitrarily small, in other words, that the difference quotient is locally bounded. The exact value of the derivative at $a$ also doesn't matter.






                        share|cite|improve this answer

























                          up vote
                          1
                          down vote













                          The heart of your argument is basically the following. Give me some $epsilon$, and I have to find some $delta$ so that $|f(x)-f(a)|<epsilon$ when $|x-a|<delta$. To start with, I'll just take any $delta$, it doesn't matter. Now, within the range $[a-delta, a+delta]$, the function $xtofrac{f(x)-f(a)}{x-a}$ is bounded, because it's convergent at $a$ and convergent functions are locally bounded - this is a theorem which you essentially spend the first couple of lines of your proof re-proving by way of the value $epsilon'$. So say $lvertfrac{f(x)-f(a)}{x-a}rvert<M$ on the interval $[a-delta, a+delta]$. Well then certainly $|f(x)-f(a)|<Mdelta$, and by choosing a sufficiently small $delta$ we can make that less than $epsilon$ (since the $M$ only gets smaller as $delta$ gets smaller).



                          If you notice, in your own proof, the $epsilon'$ is a bit pointless. Since at the end you're just going to say "and now take $delta$ as small as is necessary to make this true", you may as well initially take $epsilon'=10^{100}$. All that matters is that $epsilon'$ is finite and that $delta$ can be made arbitrarily small, in other words, that the difference quotient is locally bounded. The exact value of the derivative at $a$ also doesn't matter.






                          share|cite|improve this answer























                            up vote
                            1
                            down vote










                            up vote
                            1
                            down vote









                            The heart of your argument is basically the following. Give me some $epsilon$, and I have to find some $delta$ so that $|f(x)-f(a)|<epsilon$ when $|x-a|<delta$. To start with, I'll just take any $delta$, it doesn't matter. Now, within the range $[a-delta, a+delta]$, the function $xtofrac{f(x)-f(a)}{x-a}$ is bounded, because it's convergent at $a$ and convergent functions are locally bounded - this is a theorem which you essentially spend the first couple of lines of your proof re-proving by way of the value $epsilon'$. So say $lvertfrac{f(x)-f(a)}{x-a}rvert<M$ on the interval $[a-delta, a+delta]$. Well then certainly $|f(x)-f(a)|<Mdelta$, and by choosing a sufficiently small $delta$ we can make that less than $epsilon$ (since the $M$ only gets smaller as $delta$ gets smaller).



                            If you notice, in your own proof, the $epsilon'$ is a bit pointless. Since at the end you're just going to say "and now take $delta$ as small as is necessary to make this true", you may as well initially take $epsilon'=10^{100}$. All that matters is that $epsilon'$ is finite and that $delta$ can be made arbitrarily small, in other words, that the difference quotient is locally bounded. The exact value of the derivative at $a$ also doesn't matter.






                            share|cite|improve this answer












                            The heart of your argument is basically the following. Give me some $epsilon$, and I have to find some $delta$ so that $|f(x)-f(a)|<epsilon$ when $|x-a|<delta$. To start with, I'll just take any $delta$, it doesn't matter. Now, within the range $[a-delta, a+delta]$, the function $xtofrac{f(x)-f(a)}{x-a}$ is bounded, because it's convergent at $a$ and convergent functions are locally bounded - this is a theorem which you essentially spend the first couple of lines of your proof re-proving by way of the value $epsilon'$. So say $lvertfrac{f(x)-f(a)}{x-a}rvert<M$ on the interval $[a-delta, a+delta]$. Well then certainly $|f(x)-f(a)|<Mdelta$, and by choosing a sufficiently small $delta$ we can make that less than $epsilon$ (since the $M$ only gets smaller as $delta$ gets smaller).



                            If you notice, in your own proof, the $epsilon'$ is a bit pointless. Since at the end you're just going to say "and now take $delta$ as small as is necessary to make this true", you may as well initially take $epsilon'=10^{100}$. All that matters is that $epsilon'$ is finite and that $delta$ can be made arbitrarily small, in other words, that the difference quotient is locally bounded. The exact value of the derivative at $a$ also doesn't matter.







                            share|cite|improve this answer












                            share|cite|improve this answer



                            share|cite|improve this answer










                            answered Nov 19 at 20:08









                            Jack M

                            18.4k33778




                            18.4k33778






















                                up vote
                                1
                                down vote













                                Not quite right. One of the issues is:




                                I define $epsilon'$ as $frac epsilon delta $




                                As it stands, this doesn't make sense, because you started the proof by taking an arbitrary $epsilon'$. Perhaps you mean something like: Since this inequality is true for any $epsilon '$, it is true in particular for $frac epsilon delta$ ...



                                But this doesn't quite make sense either; What's the $delta$ on the right-hand side? This $delta $ may depend on the $epsilon '$ you initially choose.





                                Try this alternative:



                                For any $epsilon> 0$ there is a $delta' > 0$, such that whenever $0 < |x-a| < delta'$, one has $left|frac{f(x)-f(a)}{x-a}right| < |f'(a)| + epsilon$. In particular, there is a $delta_0>0$ such that $left|frac{f(x)-f(a)}{x-a}right| < |f'(a)| + 1$



                                Now, choose $delta = min left(delta_0, frac 1 {|f'(a)+1|}right) $



                                Can you complete the proof from here?






                                share|cite|improve this answer























                                • I've made a few edits to the ending paragraphs to make it a bit easier to understand what I'm trying to say.
                                  – Star Platinum ZA WARUDO
                                  Nov 19 at 15:44















                                up vote
                                1
                                down vote













                                Not quite right. One of the issues is:




                                I define $epsilon'$ as $frac epsilon delta $




                                As it stands, this doesn't make sense, because you started the proof by taking an arbitrary $epsilon'$. Perhaps you mean something like: Since this inequality is true for any $epsilon '$, it is true in particular for $frac epsilon delta$ ...



                                But this doesn't quite make sense either; What's the $delta$ on the right-hand side? This $delta $ may depend on the $epsilon '$ you initially choose.





                                Try this alternative:



                                For any $epsilon> 0$ there is a $delta' > 0$, such that whenever $0 < |x-a| < delta'$, one has $left|frac{f(x)-f(a)}{x-a}right| < |f'(a)| + epsilon$. In particular, there is a $delta_0>0$ such that $left|frac{f(x)-f(a)}{x-a}right| < |f'(a)| + 1$



                                Now, choose $delta = min left(delta_0, frac 1 {|f'(a)+1|}right) $



                                Can you complete the proof from here?






                                share|cite|improve this answer























                                • I've made a few edits to the ending paragraphs to make it a bit easier to understand what I'm trying to say.
                                  – Star Platinum ZA WARUDO
                                  Nov 19 at 15:44













                                up vote
                                1
                                down vote










                                up vote
                                1
                                down vote









                                Not quite right. One of the issues is:




                                I define $epsilon'$ as $frac epsilon delta $




                                As it stands, this doesn't make sense, because you started the proof by taking an arbitrary $epsilon'$. Perhaps you mean something like: Since this inequality is true for any $epsilon '$, it is true in particular for $frac epsilon delta$ ...



                                But this doesn't quite make sense either; What's the $delta$ on the right-hand side? This $delta $ may depend on the $epsilon '$ you initially choose.





                                Try this alternative:



                                For any $epsilon> 0$ there is a $delta' > 0$, such that whenever $0 < |x-a| < delta'$, one has $left|frac{f(x)-f(a)}{x-a}right| < |f'(a)| + epsilon$. In particular, there is a $delta_0>0$ such that $left|frac{f(x)-f(a)}{x-a}right| < |f'(a)| + 1$



                                Now, choose $delta = min left(delta_0, frac 1 {|f'(a)+1|}right) $



                                Can you complete the proof from here?






                                share|cite|improve this answer














                                Not quite right. One of the issues is:




                                I define $epsilon'$ as $frac epsilon delta $




                                As it stands, this doesn't make sense, because you started the proof by taking an arbitrary $epsilon'$. Perhaps you mean something like: Since this inequality is true for any $epsilon '$, it is true in particular for $frac epsilon delta$ ...



                                But this doesn't quite make sense either; What's the $delta$ on the right-hand side? This $delta $ may depend on the $epsilon '$ you initially choose.





                                Try this alternative:



                                For any $epsilon> 0$ there is a $delta' > 0$, such that whenever $0 < |x-a| < delta'$, one has $left|frac{f(x)-f(a)}{x-a}right| < |f'(a)| + epsilon$. In particular, there is a $delta_0>0$ such that $left|frac{f(x)-f(a)}{x-a}right| < |f'(a)| + 1$



                                Now, choose $delta = min left(delta_0, frac 1 {|f'(a)+1|}right) $



                                Can you complete the proof from here?







                                share|cite|improve this answer














                                share|cite|improve this answer



                                share|cite|improve this answer








                                edited Nov 19 at 23:50









                                miracle173

                                7,31222247




                                7,31222247










                                answered Nov 19 at 15:18









                                Praneet Srivastava

                                762516




                                762516












                                • I've made a few edits to the ending paragraphs to make it a bit easier to understand what I'm trying to say.
                                  – Star Platinum ZA WARUDO
                                  Nov 19 at 15:44


















                                • I've made a few edits to the ending paragraphs to make it a bit easier to understand what I'm trying to say.
                                  – Star Platinum ZA WARUDO
                                  Nov 19 at 15:44
















                                I've made a few edits to the ending paragraphs to make it a bit easier to understand what I'm trying to say.
                                – Star Platinum ZA WARUDO
                                Nov 19 at 15:44




                                I've made a few edits to the ending paragraphs to make it a bit easier to understand what I'm trying to say.
                                – Star Platinum ZA WARUDO
                                Nov 19 at 15:44


















                                draft saved

                                draft discarded




















































                                Thanks for contributing an answer to Mathematics Stack Exchange!


                                • Please be sure to answer the question. Provide details and share your research!

                                But avoid



                                • Asking for help, clarification, or responding to other answers.

                                • Making statements based on opinion; back them up with references or personal experience.


                                Use MathJax to format equations. MathJax reference.


                                To learn more, see our tips on writing great answers.





                                Some of your past answers have not been well-received, and you're in danger of being blocked from answering.


                                Please pay close attention to the following guidance:


                                • Please be sure to answer the question. Provide details and share your research!

                                But avoid



                                • Asking for help, clarification, or responding to other answers.

                                • Making statements based on opinion; back them up with references or personal experience.


                                To learn more, see our tips on writing great answers.




                                draft saved


                                draft discarded














                                StackExchange.ready(
                                function () {
                                StackExchange.openid.initPostLogin('.new-post-login', 'https%3a%2f%2fmath.stackexchange.com%2fquestions%2f3005021%2fproof-verification-differentiability-implies-continuity%23new-answer', 'question_page');
                                }
                                );

                                Post as a guest















                                Required, but never shown





















































                                Required, but never shown














                                Required, but never shown












                                Required, but never shown







                                Required, but never shown

































                                Required, but never shown














                                Required, but never shown












                                Required, but never shown







                                Required, but never shown







                                Popular posts from this blog

                                Biblatex bibliography style without URLs when DOI exists (in Overleaf with Zotero bibliography)

                                ComboBox Display Member on multiple fields

                                Is it possible to collect Nectar points via Trainline?